Solve each triangle. Round your answers to the nearest tenth.
show work If possible ​

Solve Each Triangle. Round Your Answers To The Nearest Tenth.show Work If Possible

Answers

Answer 1
Answer:

∠ABC = 76°

BC = 20.1

CA = 28.0

Step-by-step explanation:

Solving the triangle means finding all unknown angles and sides of the triangle.

(i) Two of the angles (∠BCA = 60° and ∠CAB  = 44°) are given. To find the third angle (∠ABC), use one of the theorems stating that the sum of angles of a triangle is equal to 180°.

Therefore, the sum of angles of the triangle ABC is 180°. i.e

∠ABC + ∠BCA + ∠CAB = 180°

=> ∠ABC + 60° + 44° = 180°

=> ∠ABC + 104° = 180°

=> ∠ABC = 180° - 104°

=> ∠ABC = 76°

(ii) One side (BA) of the triangle is given. To get the other sides, we use the sine rule as follows;

=> [tex]\frac{sin60}{25} = \frac{sin44}{BC} = \frac{sin76}{CA}[/tex]

=> [tex]\frac{sinBCA}{BA} = \frac{sinCAB}{BC} = \frac{sinABC}{CA}[/tex]

Substitute the necessary values

[tex]\frac{sin60}{25} = \frac{sin44}{BC} = \frac{sin76}{CA}[/tex]      ---------------------(ii)

(a) To get side BC, use the first two terms of equation (ii)

[tex]\frac{sin60}{25} = \frac{sin44}{BC}[/tex]

Cross multiply

BC x sin 60 = 25 x sin 44

BC x 0.8660 = 25 x 0.6947

0.8660 x BC = 17.3675

BC = [tex]\frac{17.3675}{0.8660}[/tex]

BC = 20.05

=> BC = 20.1 to the nearest tenth

(b) To get CA, use any two terms of equation (ii). Using the first and third terms, we have;

[tex]\frac{sin60}{25} = \frac{sin76}{CA}[/tex]

Cross multiply

CA x sin 60 = 25 x sin 76

CA x 0.8660 = 25 x 0.9703

0.8660 x CA = 24.2575

CA = [tex]\frac{24.2575}{0.8660}[/tex]

CA = 28.01

=> CA = 28.0 to the nearest tenth


Related Questions

Find the value of x in the isosceles triangle shown below​

Answers

Answer:

x²+4² =8²

x²+16 =64

x² =64-16.

x² =48

x. =√48

answer is option D

Answer:

D

Step-by-step explanation:

The segment x is a perpendicular bisector and divides the triangle into 2 right triangles.

Using Pythagoras' identity in either of the 2 right triangles, then

x² + 4² = 8²

x² + 16 = 64 ( subtract 16 from both sides )

x² = 48 ( take the square root of both sides )

x = [tex]\sqrt{48}[/tex] → D

Two containers, A and B begin with equal volumes of liquid.
120 ml is then poured from A to B.
А
B
Container B now contains 4 times as much liquid as A.
Find the volume of liquid left in container A at the end.
A
A
B.
B
Volume left in container A=
20
ml

Answers

Answer:

volume left in A is 80ml

Step-by-step explanation:

let original volume in both be x ml

after pouring from A to B

vol in A = x - 120

vol in B = x + 120

now it is given that

vol in B = 4 times vol in A

implies

[tex]x + 120 = 4 \times (x - 120) \\ x + 120 = 4x - 480 \\ 3x = 600 \\ x = 200[/tex]

hence original volume in A is 200ml. after pouring in B it is 200-120 = 80 ml

Suzie bought 17.75 lbs of flour. If the price for 1lb is $19.68, calculate the total amount she paid to the shopkeeper

Answers

17.75 x 19.68 = $349.32 is the answer
17.75 pounds of flour
( excluding the .75)
17 x 19.68 = $334.56
.75 = 3/4
19.68 / 4 = $4.92
($4.92 for every quarter of a pound)
4.92 x 3 = $14.76 ( .75 = 3/4)
334.56 + 14.76 = $349.32
CHECK
17.75 x 4 = 71 ( Figure out how many fourths make 17.75)
71 x 4.92 = $349.32

She paid the shopkeeper $349.32

what do you think the soultion is to the queation. I need help???​

Answers

Answer:

w = 16, x = 11, y = 6, z = 7.5

Step-by-step explanation:

The scale factor is the ratio of corresponding sides, image to original.

scale factor = [tex]\frac{6}{8}[/tex] = [tex]\frac{3}{4}[/tex]

Then by proportion

[tex]\frac{12}{w}[/tex] = [tex]\frac{3}{4}[/tex] ( cross- multiply )

3w = 48 ( divide both sides by 3 )

w = 16

----------------

[tex]\frac{x}{12}[/tex] = [tex]\frac{3}{4}[/tex]

4x = 36

x = 9

-------------------

[tex]\frac{4.5}{y}[/tex] = [tex]\frac{3}{4}[/tex]

3y = 18

y = 6

----------------------------

[tex]\frac{z}{10}[/tex] = [tex]\frac{3}{4}[/tex]

4z = 30

z = 7.5

an article cost $1424 it may be purchased by depositing $560 and making monthly payment of 48 how many monthly are required to complete payment?

Answers

Answer:

18 monthly payments are required to complete payment

Step-by-step explanation:

We start with our $1,424.

Then subtract $560 from the original number above.

Leaving us with $864.

We then divide the $864 by $48 and we end up with 18.

So our answer is 18 monthly payments are required to complete payment.

I need help for this math question ASAP

A line segment joins points P(–1, –7) and Q(7, 3). Determine the length of PQ to 1 decimal place.

Answers

I’m just doing this for (-7,-1)

How many times does
1/8 go into 1/4

Answers

Answer:

One eighth is one part of eight equal sections. Two eighths is one quarter and four eighths is a half. It's easy to split an object, like a cake, into eighths if you make them into quarters and then divide each quarter in half

Which is NOT a way to state the meaning of the expression c+5?

Answers

I need to see the graph

What is the equation of the line that passes through point (3, −1) and has a slope of 2?

Answers

Answer:

equation y = 3x + b.

Step-by-step explanation:  

To come up with the equation of a line in slope intercept form, we need the slope and the y-intercept. First, substitute the given slope in for m. We get the equation y = 3x + b.

Tomato juice comes in three different sized cans. The red can contains 6 cups, the blue can contains 2 pints, and the yellow can contains 32 fluid ounces. Part A Which can holds the most tomato juice? Enter red, blue, or yellow in the answer box.

Answers

Answer:

Red can

Step-by-step explanation:

Tonato juice come in different sizes

The red can contains 6 cups

The blue can contains 2 pints

The yellow can contains 32 fluid ounces

Therefore the can that holds the highest juice is the red can with 6 cups if tomato juice

1. In the diagram below, segment AB is tangent to circle C at point D and
AC LBC. If m

Answers

Answer:

∠ DCB = 28°

Step-by-step explanation:

∠ ACD = 180° - (90 + 28)° = 180° - 118° = 62° , then

∠ DCB = 90° - 62° = 28°

There are 5 red marbles, 15 blue marbles,
reach in and randomly draw two marbles without replacing the first, what is the
probability that both will be blue?

Answers

Answer:

Probability is 0.552, or about 55%

Step-by-step explanation:

first there are 20 marbles in total: odds are 15/20 = 3/4

Then, there are 19 marbles in total : odds are 14/19

So the odds of both of them being blue are: 3/4 * 14/19 = 0.552 (rounded to the thousandths place)

Please give brainliest thanks!

can someone PLEASE help me with this!?! its due tonight

explaining your answer = brainliest

what is the perimeter of the square?

Answers

Answer:

24

Step-by-step explanation:

6*4

The value of a car is deprecting at the rate of 12% Find the correct value of car when cost of car was rs 350000 last year

Answers

Answer:

The cost of the car is $ RS 308,000.

Step-by-step explanation:

Given that the value of a car is deprecting at the rate of 12%, to find the correct value of car when cost of car was $ RS350,000 last year, the following mathematical calculation must be performed:

100 - 12 = 88

88/100 = 0.88

350,000 x 0.88 = X

308,000 = X

Therefore, the cost of the car is $ RS 308,000.

5.0×10

1


1.50×10

5




= ?

help please last day of school

Answers

Answer:

5.0x10=50

1.50x10=15

Step-by-step explanation:

calculator

Lauren is tutoring students at the library on Saturday. If she is at the library for a total of 6 hours and she helps each
student, one at a time, for 7 of an hour, how many students does she tutor?
3

Answers

Answer:

Lauren tutors 42 students in total

Step-by-step explanation:

She tutors for 6 hours and she teaches 7 students an hour , so we just have to multiply 7 by 6 which is 42

Hope it helps:)

2+2+2+2-2×2-2+2=? . (16-14-12-24)​

Answers

Answer:

12 I think

I'm not sure

Byeee

Answer:

4

Step-by-step explanation:

2+2+2+2-2×2-2+2 = 4

HOPE IT HELPS YOU ✌️

Please Help I'll give brainliest

Answers

Answer:

259.8

Step-by-step explanation:

259.8. Is the answer

find the percent increase is admission to the amusement park is raised from $70 to $80.

Answers

Answer:

14.286% increase

Step-by-step explanation:

Since the admission to the amusement park increased by $10 we need to find what percentage of $70 equals $10.

We can find this percentage by multiplying 10 (the difference in money) by 100 (a hundred percent), and dividing that by 70 (the initial cost)

[tex]10*100/70=14.286[/tex]

the sixth graders rasied money to fund a feild trip. the five classes raised $42, $51, $38, $49, and $40. what is the median on the ammount of money raised?

a. $36
b. $42
c. $40
d. $13

Answers

Answer; 38


Explanation: it is the number between 51 and 49

Answer:

The median is $42, or B.

Step-by-step explanation:

Our numbers are :

42, 51, 38, 49, and 40.

The first step to finding the median is ordering the numbers from least to greatest.

38, 40, 42, 49, 51.

Now we find the number perfectly in the middle - that's 42, between 49 and 40. The median is $42, or B.

Please mark brainliest!

In a survey, 15 high school students said they could drive and 15 said they could not. Out of 60 college students surveyed, 30 said they could drive. Micah concluded that knowing that a person is in college means they are more likely to drive. Is Micah’s conclusion correct? Explain.

Answers

Answer:

No.

Step-by-step explanation:

#1, the sample size is not indicative of a proper high school population, unless of all there is a small amount of students in the high school (or even only 30 students in it). On a individual basis, it can vary widely, and so data analysis such as this is not correct. The data size is skewed as well for the college, especially as college campuses typically house more students then usual. To only ask 60 college students also skew the data gotten. Another sample error may be locational error, in which the location of the schools may warrant the need for a driver license, or if it is not needed, as well as the distance of the student's house and/or where needed facilities are to accommodate a student's needs and desires.

If we were to set all these problems aside and focus more on the question, 15 high school students said they could drive, and 15 said they could not, which makes the sample size of 30 high school students in all. 15 say they can drive, therefore:

15/30 = 1/2, or there is a 50% chance of obtaining a student that can drive.

On the other hand, out of 60 college students surveyed, 30 said they can drive, therefore:

30/60 = 1/2, or there is a 50% chance of obtaining a student that can drive.

∴ Micah's prediction is incorrect in obtaining a college student that can drive more easily then obtaining one that can drive in a high school.

However, remember that too small of a data size can lead to a heavy skew, and it will not represent reality as well.

~

Answer:

Sample Response: No, Micah is not correct. Since there are equal numbers in the group that drive and the group that doesn’t drive, the relative frequency for each is 50%. Because the relative frequencies are the same, there is no association between the variables.

Step-by-step explanation:

Got this from Edg 2021

Any help is appreciated!!!!!!

Answers

Answer:

d

Step-by-step explanation:

ddddddddddddddddddddddddddfffffdff

I think the answer Is B but i am not sure

You are taking a survey on your experience at Taco Bell. For the first three questions you can
answer Below Average, Average, or Above Average for each question. The last five questions
you can respond with either Agree or Disagree. How many total outcomes are there for this
survey? *

Answers

19 out come
9 for the first 3 questions
10 for the last 5 questions

Hope this helps ;)

The total outcomes for this survey are 864.

What are the outcomes of the set?

An elementary event is one with only one possible consequence. The sample space of an experiment is the event that contains all feasible results. A single consequence may appear in a variety of circumstances.

Given, You are responding to a poll about Taco Bell. You can select Below Average, Average, or Above Average for each of the first three questions. You may choose to agree or disagree with the past five questions.

Here, we utilize the combinations formula to determine how many outcomes there are in this survey.

Three questions have three options, while five questions have two.

Hence,

total outcomes = 3³ * 2⁵ = 27 * 32 = 864

Therefore, There are 864 total responses to this survey.

Learn more about the outcomes of the set here:

https://brainly.com/question/28353187

#SPJ2

Can someone help I’m confused

Answers

Answer:

D and E

Step-by-step explanation:

Let's double check:

D. 9 ÷ 3 = 3

E. 4 × 9 = 36

---------------------------------------------------------------------------------------------------------------

Have a great summer :)

Answer:

the answers would be A, D, and E

Step-by-step explanation:

just replace the Cs with the number 9 and solve from there.

for example c/3=3 would say 9/3=3

Find the equation in terms of x of the line through the points (-4,-6) and (1,9)

Answers

Answer:

y = 3x + 6

Step-by-step explanation:

y2 - y1 / x2 - x1

9 - (-6) / 1 - (-4)

15/5

= 3

y = 3x + b

9 = 3(1) + b

9 = 3 + b

6 = b

Rectangle, Square, or Rhombus?

Answers

Answer:

maybe rhombus

Step-by-step explanation:

don't take my word for it

Answer:

I think Rhombus. I am not so sure.

Step-by-step explanation:

Have a nice day! :-)

HELP ME NEED HELP NOW

Answers

Answer:

the product X Y is constant so the relationship is not an inverse variation.

Step-by-step explanation:

plzzzzzzzzz guysssss im failinggggg

Answers

Step-by-step explanation:

a) Given,

Total marks in English = 90

Marks scored by James = 60

Therefore,

Percentage of marks scored

[tex] = \frac{60}{90} \times 100[/tex]

= 66.67% (approx. )

Then,

Total marks in Science = 150

Marks scored by James = 78

Therefore,

Percentage of marks scored

[tex] = \frac{78}{150} \times 100[/tex]

= 52%

At last,

Total marks in Maths = 120

Marks scored by James = 75

Therefore,

Percentage of marks scored

[tex] = \frac{75}{120} \times 100[/tex]

= 62.50%

b) In English test James scored highest percentage.

Find x round your answer to the nearest tenth of a degree PLZ HELP ASAP BRAINLIEST!!!!!!!

Answers

Answer:

45.9˚

Step-by-step explanation:

use trig formulas

23 and 16 correspond to the hyp and adj sides to angle x.

adj/hyp is cos so use the cos formula.

cos x = 16/23

arccos --> inverse cos

x = arccos(16/23)

put it in a calculatior and round to nearest 10th.

45.92079....

= 45.9˚

Answer:

Step-by-step explanation:

if the needle below is spun twice and the letters are recorded both times , which of the following would be the probability it would land on letters A and C (in either order)?

(1) 1/3
(2)2/3
(3)2/9
(4)5/6|

Answers

2/3 is the correct answer
Other Questions
help...................... What does a health community look like? KJ Pharma issues new bonds to fund an acquisition. The face value of the bond is $100 and annual coupon is 6.5%. Further, this bond matures in 20 years and is issued at a price of $105. Assuming KJ Pharma's tax rate is 30%, what is its After-Tax Cost of Debt What is the surface area of the solid that this net can form?8 mm25 mm8 mm5 mm5 mm8 mm8 mm5 mm5 mm8 mm8 mm25 mmO 730 square millimetersO 875 square millimetersO 1,000 square millimetersO 1,444 square millimeters Select the correct answer.What is the factored form of this expression?-12x+36.(x - 12)(x-3)O B. (x - 6)^2OC. (x + 6)^2OD. (x-6)(x+6) Did the Tea Act lead to the American Revolution? If yes, explain why. es una fuente de energa no contaminante A sculptor needs to lift a piece of marble its a cuboid with dimensions 1 by 0.2 by 0.5 marble has a density of 2.7 whats the mass in kg You have decided to buy a used car. The dealer has offered you two options: (FV of $1, PV of $1, FVA of $1, and PVA of $1) (Use the appropriate factor(s) from the tables provided.) Pay $620 per month for 20 months and an additional $12,000 at the end of 20 months. The dealer is charging an annual interest rate of 24%. Make a one-time payment of $16,864, due when you purchase the car. 1-a. Determine how much cash the dealer would charge in option (a). (Round your answer to 2 decimal places.) 1-b. In present value terms, which offer is clearly a better deal Scientists found that a certain particle of dust doubles every 30 minutes. If the scientists tests this out with 21 grams of the specific dust, how many grams would they have after 2 days? help me pls that would be chilly A company that lacks a competitively powerful stand-alone resource Multiple choice question. will be unable to develop competencies through cross-functional efforts. may attempt to develop a competitive advantage through resource bundling. must continue searching for a resource that passes five tests of competitive power. is likely to be a failure in the marketplace. If you could somehow travel inside an atom and look around, what part of the atom would you want to look at? What would you want to understand better as a result of your "travels"? Write your answers in 2-3 paragraphs. Https://imgur.com/a/n3PRcMY help please jfisaljfdlsajflsd D. Short answer questions:a. Compare the monopolys price and quantity with that of aperfectly competitive firms. Which is better for the consumer?b. How can government regulation of a monopoly increase theallocative efficiency of the monopoly? An intelligence signal is amplified by a 65% efficient amplifier before being combined with a 250W carrier to generate an AM signal. If it is desired to operate at 50% modulation, what must be the dc input power to the final intelligence signal amplifier introduction: connect each word with the phrases that best defines it. The following table shows the probability of rolling the numbers 1 through 7on a fair 7-sided die.1234567P(X) 1/71/71/71/71/71/71/7What is the standard deviation of the random variable X, "the number thatcomes up on the die"?A. 1.708OB. 3.745O C. 2.917D. 3.5E. 2.0 what is function of spacebar